2011 AMC 10A Problems/Problem 5

Revision as of 19:05, 10 February 2011 by Aopsmath1012 (talk | contribs) (Created page with 'Without loss of generality, let there be 1 fifth grader. It follows that there are 2 fourth graders and 4 third graders. We have <math>\frac{(1)(10)+(2)(15)+(4)(12)}{1+2+4} = \f…')
(diff) ← Older revision | Latest revision (diff) | Newer revision → (diff)

Without loss of generality, let there be 1 fifth grader. It follows that there are 2 fourth graders and 4 third graders. We have $\frac{(1)(10)+(2)(15)+(4)(12)}{1+2+4} = \frac{88}{7}$.